Dar sentido a las relaciones canónicas de anticonmutación para los espinores de Dirac

Cuando se hace QFT escalar, normalmente se imponen las famosas 'relaciones de conmutación canónicas' en el campo y el momento canónico:

[ ϕ ( X ) , π ( y ) ] = i d 3 ( X y )
en tiempos iguales ( X 0 = y 0 ). Es fácil (aunque tedioso) comprobar que esto implica una relación de conmutación para los operadores de creación/aniquilación
[ a ( k ) , a ( k ) ] = ( 2 π ) 3 2 ω d 3 ( k k )

Al considerar el campo de Dirac (espinor), es habitual (ver, por ejemplo, la página 107 de las notas de Tong o el libro de Peskin & Schroeder) proceder de manera análoga (reemplazando los conmutadores con anticonmutadores, por supuesto). Nosotros postulamos

{ Ψ ( X ) , Ψ ( y ) } = i d 3 ( X y )
y, a partir de ellos, derivar las relaciones usuales para los operadores de creación/aniquilación.

Siempre acepté esto y creí en los cálculos presentados en las fuentes mencionadas anteriormente, pero de repente me encuentro con dudas: ¿Estas relaciones tienen algún sentido para el campo de Dirac? Desde Ψ es un espinor de 4 componentes, realmente no veo cómo uno puede tener sentido de la ecuación anterior: ¿No es Ψ Ψ a 4 × 4 matriz, mientras Ψ Ψ es un numero?! ¿Tenemos que hacer el cálculo (spinor-) componente por componente? Si este es el caso, entonces creo que veo algunas dificultades (en los cálculos habituales se necesita una identidad que depende de que los 4 espinores sean realmente 4 espinores). ¿Se evitan de alguna manera? Una explicación detallada sería muy apreciada.

Como seguimiento, considere lo siguiente: Generalmente se encuentran términos como este en el cálculo:

tu a a tu tu a a tu
Incluso si se acepta que una ecuación como { Ψ , Ψ } tiene sentido, la mayoría de las fuentes simplemente "tiran del tu ,   tu fuera de los conmutadores para obtener (anti) conmutadores de solo los operadores de creación/aniquilación. ¿Cómo se justifica esto?

EDITAR : Me acabo de dar cuenta de que la relación de conmutación correcta quizás sustituye Ψ con Ψ ¯ (esto puede evitar cualquier problema que surja en un cálculo por componentes). Por favor, siéntase libre de usar cualquiera en una respuesta.

No estoy seguro de haber entendido tu punto. En el caso de los espinores de Dirac las reglas de anticonmutación funcionan por componentes (ver por ejemplo Peskin & Schroeder p.58):
{ ψ α ( X ) , ψ β ( y ) } = d α β d 3 ( X y )
entonces tienes una matriz 4x4 en RHS. ¿Era esta tu pregunta?
@glance ¿Los cálculos son por componentes? Esto no es obvio para mí (las 'fuentes estándar' ciertamente no muestran explícitamente estos índices), aunque ya lo mencioné como una posibilidad. Si puede realizar el cálculo de esta manera (evitando algunos problemas menores que preveo) y publicarlo como respuesta, ¡estaría más que feliz de aceptarlo!
Ver Peskin y Schroeder eq. 3.86 para verificar que la relación es por componentes.
@joshphysics maldita sea , estaba mirando 3.89 y no vi ningún componente, y me asusté. Supongo que eso lo resuelve...
¡Alguien debería publicar eso como respuesta! (FWIW llegué a la misma conclusión de las notas de Tong)
@Danu, por supuesto, pero ¿a qué cálculo te refieres exactamente? ¿La derivación de las reglas de conmutación para la creación/operadores de las reglas de conmutación canónicas (su primera ecuación) o las relaciones de (anti)conmutación de tiempo igual (su segunda)? Además, creo que la referencia en P&S es 3.102, en 3.89 muestran que los conmutadores no funcionan para los espinores de Dirac (al menos en mi edición)
@glance Creo que debería recibir los honores y solo incluir una referencia a P&K para corroborar su comentario. De acuerdo en que 3.89 es inapropiado.
@glance derivar el CCR para los operadores de creación/aniquilación del campo Dirac sería más que satisfactorio.
@joshphysics Por supuesto, 3.89 no es el cálculo para { , } pero es el único cálculo explícito, por eso estaba buscando allí;)
Relacionado: physics.stackexchange.com/q/17893/2451 y enlaces allí.

Respuestas (1)

Por lo general, se parte del CCR para los operadores de creación/aniquilación y se derivan de allí las reglas de conmutación para los campos. Sin embargo, uno puede comenzar desde cualquiera (ver por ejemplo aquí sobre esto). Supongamos que queremos partir de las reglas de anticonmutación de igual tiempo para un campo de Dirac ψ α ( X ) :

(1) { ψ α ( X ) , ψ β ( y ) } = d α β d 3 ( X y ) ,
dónde ψ α ( X ) tiene una expansión de la forma
(2) ψ α ( X ) = d 3 pag ( 2 π ) 3 2 mi pag s { C s ( pag ) [ tu s ( pag ) ] α mi i pag X + d s ( pag ) [ v s ( pag ) ] α mi i pag X }
o más concisamente
ψ ( X ) = d pag ~ ( C pag tu pag mi i pag X + d pag v pag mi i pag X ) ,

y queremos derivar el CCR para los operadores de creación/aniquilación:

(3) { a s ( pag ) , a s ( q ) } = ( 2 π ) 3 ( 2 mi pag ) d s s d 3 ( pag q ) .
Para ello, queremos expresar a s ( pag ) en términos de ψ ( X ) . Tenemos:
(4) a s ( k ) = i tu ¯ s ( k ) d 3 X [ mi i k X 0 ψ ( X ) ψ ( X ) 0 mi i k X ] = i tu ¯ s ( k ) d 3 X mi i k X 0 ψ ( X )
(5) a s ( k ) = i tu ¯ s ( k ) d 3 X [ mi i k X 0 ψ ( X ) ψ ( X ) 0 mi i k X ] = i tu ¯ s ( k ) d 3 X mi i k X 0 ψ ( X )
que puede verificar tirando de la expansión (2) en (4) y (5) . Tenga en cuenta que estos valen para cualquier X 0 en el RHS.

Ahora solo tienes que insertar en el anticonmutador en el LHS de (3) estas expresiones y usar (1) (puedo ampliar un poco este cálculo si lo necesitas).

la mayoría de las fuentes simplemente "tiran del tu , tu fuera de los conmutadores para obtener (anti) conmutadores de solo los operadores de creación/aniquilación. ¿Cómo se justifica esto?

Hay una gran diferencia entre un espinor de polarización tu y un operador de creación/destrucción C , C .

Para polarización fija s y el impulso pag , tu s ( pag ) es un espinor de cuatro componentes, lo que significa que tu s ( pag ) α C para cada α = 1 , 2 , 3 , 4 . Por el contrario, para polarización fija s y el impulso pag , C s ( pag ) es un operador en el espacio de Fock . No solo un número, lo que hace que las preguntas sobre (anti)conmutadores sean significativas.

Por "doble flecha" quieres decir o dos flechas derechas una encima de la otra?
Me refiero a la flecha que apunta a ambos lados... el símbolo utilizado para indicar una derivada a la derecha menos una derivada a la izquierda: en a ¯ m b a m b ( m a ) b el símbolo que normalmente se usaría en lugar de la barra en ¯
\overset{\leftrightarrow}{\partial}
Una flecha de una sola línea que apunta en ambos sentidos es \leftrightarrow: . Puedes poner eso encima usando \overset{up}{down}: m .